Questions tagged [forcing]

Forcing is a method first used to prove the continuum hypothesis is independent of the classical axioms of set theory

159 questions with no upvoted or accepted answers
Filter by
Sorted by
Tagged with
27 votes
0 answers
1k views

Where do uncountable models collapse to?

Suppose $T$ is a complete first-order theory (in an finite, or at worst countable, language). Given any model $\mathcal{M}\models T$ of cardinality $\kappa$, we can ask whether $\mathcal{M}$ can be ...
Noah Schweber's user avatar
19 votes
0 answers
881 views

What examples of existence forcing proofs are there?

Forcing proofs tend to be fairly constructive, in the sense that if I claim that there is a forcing that does something, I usually prove this by constructing that forcing. There are only a handful of ...
Asaf Karagila's user avatar
  • 37.9k
18 votes
0 answers
899 views

Souslin trees and weakly compact cardinals

In Souslin trees on the first inaccessible cardinal it is asked if it is consistent that there are no $\kappa-$Souslin trees at the least inaccessible cardinal $\kappa$. In this question I would like ...
Mohammad Golshani's user avatar
17 votes
0 answers
541 views

Gitik's work on Shelah's weak hypothesis

It seems that Moti Gitik has recently refuted some variants of Shelah's weak hypothesis. For this see the title and abstract of his talk at the Set Theory, Model Theory and Applications conference. I ...
Mohammad Golshani's user avatar
14 votes
0 answers
391 views

O-minimality and forcing

It is well-known that the structure $(\mathbb{R}, +, \cdot, <, 0, 1)$ is an o-minimal structure and hence the set of integers $\mathbb{Z}$ is not definable in it. In an ongoing project with Will ...
Mohammad Golshani's user avatar
14 votes
0 answers
593 views

Cohen/Random reals over intermediate models in countable support iterations

Assume that the continuum is $\aleph_2$ in our ground model $V$. Suppose that $(P_n \, : \, n \in \omega)$ is a fully supported iteration of length $\omega$. Suppose further that the factors of the ...
Stefan Hoffelner's user avatar
14 votes
0 answers
342 views

The failure of GCH al $\aleph_\omega$ by nice forcing

There are several ways to force $GCH$ below $\aleph_\omega$ and $2^{\aleph_\omega}> \aleph_{\omega+1},$ say: 1) The Gitik-Magidor's extender based forcing, see Prikry type Forcings. 2) Woodin's ...
Mohammad Golshani's user avatar
13 votes
0 answers
258 views

Generic $\mathbf{\Sigma}_3^1$-absoluteness for class forcings

In the paper "Generic Absoluteness" by Bagaria and Friedman (http://www.logic.univie.ac.at/~sdf/papers/bagfried.pdf) it is shown that in ZFC generic $\mathbf{\Sigma_3^1}$-absoluteness is false for ...
rkrapf's user avatar
  • 131
13 votes
0 answers
455 views

Adding a saturated ideal

Is it consistent that there is no $\omega_2$-saturated ideal on $\omega_1$, but one is introduced by an $\omega_2$-closed forcing? Some motivation: If $\delta$ is a Woodin cardinal, then it remains ...
Monroe Eskew's user avatar
  • 18.1k
12 votes
0 answers
491 views

Can a generic $\mathbb{R}$ have a new cardinality?

This question was asked and bountied at MSE, without success. My main question is whether, starting with a model of determinacy, a "generic $\mathbb{R}$" could be different in cardinality ...
Noah Schweber's user avatar
12 votes
0 answers
436 views

c.c.c forcing notions and adding minimal generic reals

Is the following statement consistent: ``There is no non-trivial c.c.c forcing notion adding a minimal generic real''? The question is related to Prikry's question: Is it consistent that any non-...
Mohammad Golshani's user avatar
12 votes
0 answers
719 views

Some questions about $0^{\sharp}$ and forcing over $L$

1-Let $P\in L$ be a nontrivial set forcing or even a tame class forcing (tameness in the sense of Sy Friedman; see for example his Handbook paper), and let $G$ be $P-$generic over $L$. Then it is well-...
Mohammad Golshani's user avatar
11 votes
0 answers
273 views

Can we bound $2^{\aleph_\omega}$ without pcf theory?

One of the famous applications of pcf theory is that if $\aleph_\omega$ is a strong limit cardinal then $2^{\aleph_\omega}<\aleph_{\omega_4}$. I'm curious whether any weaker result with the same ...
Noah Schweber's user avatar
11 votes
0 answers
453 views

$\Sigma^2_1$ and the Continuum Hypothesis

This is a follow up to Will Brian's answer to this recent question. In particular, quoting Brian: "In fact, Paul Larson has pointed out to me that the statement "$\phi$ and $\phi^{-1}$ are conjugate"...
Todd Eisworth's user avatar
11 votes
0 answers
264 views

Preservation of chain condition under strategically closed forcing

It is well-known that $\kappa$-closed forcing preserves $\kappa$-c.c. posets. The same argument works for $\kappa$-strategically closed forcing. Here is the definition: A poset $\mathbb P$ is $\...
Monroe Eskew's user avatar
  • 18.1k
11 votes
0 answers
356 views

On the relationship between Martin's Axiom, the countable chain condition and the Knaster property

This is a repost of a question that went unanswered on MSE We say that a poset $P$ has the Knaster property (or is Knaster) if every uncountable subset of $P$ contains an uncountable subset of ...
Alessandro Codenotti's user avatar
11 votes
0 answers
505 views

What happens when you iterate Cohen reals?

There are a few classical theorems in set theory: The finite support iteration of ccc forcing is ccc. The countable support iteration of proper forcing is proper. The finite support iteration of ...
Asaf Karagila's user avatar
  • 37.9k
11 votes
0 answers
275 views

A variant of strong ideals, is it consistent?

Is it consistent relative to large cardinals that there is a precipitous ideal on $\omega_1$ forcing a generic elementary embedding $j : V \to M \subseteq V[G]$, such that $j(\omega_1) = \omega_n^V$ ...
Monroe Eskew's user avatar
  • 18.1k
11 votes
0 answers
290 views

Preserving Jonsson cardinals

I am (still) interested in trying to characterize and describe forcings that preserve Jonsson cardinals. A cardinal $\kappa$ is a Jonsson cardinal if there is no Jonsson algebra on $\kappa$, i.e. ...
Avshalom's user avatar
  • 2,121
11 votes
0 answers
509 views

Existence of a regular subposet which collapses everything except the top cardinal

Suppose $\delta$ is an inaccessible cardinal, and $\mathbb{P}$ is the Levy Collapse $\text{Col}(\kappa, \delta)$ which adds a surjection from $\kappa \to \delta$ (for some regular $\kappa < \delta$)...
Sean Cox's user avatar
  • 2,281
10 votes
0 answers
206 views

Is $\kappa \rightarrow [\kappa]^2_3$ the same as $\kappa \rightarrow [\kappa]^2_2$ for inaccessible $\kappa$

The principle $\kappa \rightarrow [\kappa]^2_\alpha$ states that whenever we have a coloring $c:[\kappa]^2\rightarrow \alpha$ there is $H \subset \kappa$ of size $\kappa$ s.t. $|c"[H]^2|<\alpha$. ...
Jiachen Yuan's user avatar
10 votes
0 answers
267 views

Martin's Maximum implies stationary/club Chang's conjecture?

Chang's Conjecture (CC) states: for any $f: [\omega_2]^{<\omega} \to \omega_1$, there exists a set $X\subset \omega_2$ of order type $\omega_1$ such that $|f''[X]^{<\omega}|\leq \aleph_0$. ...
Jing Zhang's user avatar
  • 3,138
10 votes
0 answers
314 views

What kind of objects can code a universe?

Jensen proved that given $V\models\sf ZFC+GCH$, there is a class generic real $r$, such that $V[r]=L[r]$, and no cardinals are collapsed. We know that this can be modified such that $r$ is minimal, i....
Asaf Karagila's user avatar
  • 37.9k
10 votes
0 answers
168 views

Isomorphisms mod nonstationary

Suppose $G \subseteq \mathrm{Add}(\omega_1)$ is generic over $V$. Let $X_i = \{ \alpha : G(\alpha) = i \}$. Is it true that $P(X_0)/\mathrm{NS} \cong P(X_1)/\mathrm{NS}$?
Monroe Eskew's user avatar
  • 18.1k
10 votes
0 answers
308 views

Adjoints to forcing

Forcing over a partial order $P$ can be viewed in a category theoretic sense as constructing the presheaf topos ${\bf Set}^{P^{op}}$ over the partial order (viewed as a category) then passing through ...
Alec Rhea's user avatar
  • 8,947
10 votes
0 answers
185 views

stationary reflection in $[\kappa]^\omega$

It is well-known that the following reflection principle is consistent relative to a supercompact: For all $\kappa \geq \omega_2$ and all stationary $S \subseteq [\kappa]^\omega$, there is $X \...
Monroe Eskew's user avatar
  • 18.1k
10 votes
0 answers
431 views

On the Number of Parallel Automorphism Lines

Given a group $G$, one can define the transfinite line of iterative automorphisms of $G$ to be the following chain of the groups where $G_{\alpha+1}=Aut(G_{\alpha})$ for each ordinal $\alpha$ and the ...
Morteza Azad's user avatar
10 votes
0 answers
434 views

A new maximality principle and its consequences

Let us consider the following maximality principle: $(MP_*):$ For all uncountable regular cardinals $\kappa, 2^{<\kappa}=\kappa^{+}$ and all trees of height and size $\kappa$ are specialized. It ...
Mohammad Golshani's user avatar
10 votes
0 answers
237 views

Homogeneity of a variant of Prikry forcing

Prikry forcing is easily seen to be cone homogeneous (for any $p, q \in \mathbb{P}$, there are $p' \leq p, q' \leq q$ and an isomorphism $\Phi: \mathbb{P}/p' \simeq \mathbb{P}/q'$); in particular for ...
Mohammad Golshani's user avatar
10 votes
0 answers
288 views

When does $HOD^{V[G]} \subseteq V$?

Assume that $\mathbb{P}\in HOD$ is non-trivial. It is well-known that if $\mathbb{P}$ satisfies some homogeneity properties, then $HOD^{V[G]} \subseteq V$, where $G$ is $\mathbb{P}$-generic over $V$. ...
Mohammad Golshani's user avatar
10 votes
0 answers
254 views

History of preservation theorems in forcing theory

For my honours thesis, I am studying a general preservation theorem using a framework provided by Shelah. I am mainly concerned about revised countable support iteration of $\dot{S}$-semiproper ...
Zoorado's user avatar
  • 1,215
9 votes
0 answers
207 views

Naive way to violate $\mathsf{SCH}$ at $\aleph_\omega$

I asked this question on MSE and got a partial answer. Shamefully I still haven't figured out myself a full answer, so I would like to ask it here. The usual way to get the failure of $\mathsf{SCH}$ ...
new account's user avatar
9 votes
0 answers
310 views

Non-closed Neeman forcing

This question is something of a follow-up to this one: Iterating Neeman's forcing It regards the work of Itay Neeman, MR3201836. Neeman formulates his two-type models forcing seemingly in greater ...
Monroe Eskew's user avatar
  • 18.1k
9 votes
0 answers
242 views

Distributivity of certain infinite products

Suppose we have a sequence of posets $\{\mathbb P_n : n\in\omega\}$ such that for each $n$, $\mathbb P_{n+1}$ is $|\mathbb P_n|^+$-distributive. Is $\prod_{n>0} \mathbb P_n$ necessarily $|\mathbb ...
Monroe Eskew's user avatar
  • 18.1k
9 votes
0 answers
159 views

Strong Chains in $^{\omega_1}\omega_1$ of length $\omega_3$

In a previous question, I asked about the impact of strong chains in $^{\omega_1}\omega_1$ (e.g., sequences of functions $\langle f_\alpha:\alpha<\kappa\rangle$ in $^{\omega_1}\omega_1$ that are ...
Todd Eisworth's user avatar
9 votes
0 answers
371 views

On the role of $\diamondsuit$

The well-known axiom $\diamondsuit$ states that there is a sequence $\langle A_\alpha:\alpha<\omega_1\rangle$ (a $\diamondsuit$-sequence) of countable sets with the property that for any $A\...
Todd Eisworth's user avatar
9 votes
0 answers
225 views

Sacks property for higher cardinals

It is well known, that the Sacks forcing has the property that for any $f: \omega \rightarrow \omega$ in generic extension, one can obtain $F:\omega \rightarrow [\omega]^{<\omega}$ in ground model, ...
Ziemowit Kostana's user avatar
9 votes
0 answers
187 views

Analogue of strong stationary reflection from MM

Foreman-Magidor-Shelah proved that if Martin’s Maximum holds, then for every regular $\kappa>\omega_1$, every stationary $S \subseteq S^\kappa_\omega$ contains a club of ordertype $\omega_1$. This ...
Monroe Eskew's user avatar
  • 18.1k
9 votes
0 answers
254 views

How many iterations of inner models/generic extensions are sufficient?

Let $M=M_0$ be a ctm of ZF. If $(M_0, \ldots, M_n)$ is a sequence of ctms of ZF, where for all $i,$ either $M_{i+1}$ is an inner model of $M_i$ or a generic extension of $M_i,$ then call $M_n$ an $n$-...
Elliot Glazer's user avatar
9 votes
0 answers
345 views

Direct limits of $\sigma$-centered forcing notions

It is quite well known that Any FS (finite support) iteration of length $<\mathfrak{c}^+$ of $\sigma$-centered posets is $\sigma$-centered (see e.g. here). Now consider the following question: ...
dragoon's user avatar
  • 763
9 votes
0 answers
291 views

On an unpublished result of Magidor

In 1970th, Magidor proved the following important results: (1) Assuming the existence of a supercompact cardinal, it is consistent that $\aleph_\omega$ is strong limit and $2^{\aleph_\omega}=\aleph_{\...
Mohammad Golshani's user avatar
9 votes
0 answers
321 views

preserving saturated ideals

A reliable source made the following claim: Suppose CH there is an $\omega_2$-saturated ideal on $\omega_1$. Then this is preserved by $\mathrm{Add}(\omega_1,\omega_2)$. Question 1: How do you ...
Monroe Eskew's user avatar
  • 18.1k
9 votes
0 answers
273 views

Proving regularity properties from forcing axioms

It's well known that PFA implies projective determinacy. It's also well known that PD implies that all projective sets are Lebesgue measurable, have the Baire property, etc. Is there a direct proof ...
Haim's user avatar
  • 421
9 votes
0 answers
265 views

Which forcing types preserve the axiom of determinacy?

Do we have some rudimentary understanding of some properties that a forcing can have in order to guarantee that it doesn't violate the axiom of determinacy? To be more specific, in Which forcings ...
Asaf Karagila's user avatar
  • 37.9k
9 votes
0 answers
247 views

Consistency strength of saturated ideals on $[\lambda]^{<\kappa}$

In the Handbook of Set Theory, Foreman notes that in models constructed by Magidor from a huge cardinal, there exists a normal ideal on $[\omega_2]^{<\omega_1}$ that is $\omega_3$-saturated. Other ...
Monroe Eskew's user avatar
  • 18.1k
8 votes
0 answers
203 views

ladder system uniformization at successors of singulars

Shelah proved (paper 667) that if GCH holds and $\lambda$ is singular, then for every stationary $S \subseteq \{ \alpha < \lambda^+ : \text{cf}(\alpha) = \text{cf}(\lambda) \}$, there is a ladder ...
Monroe Eskew's user avatar
  • 18.1k
8 votes
0 answers
239 views

Topological applications of $\mathfrak{p}=\mathfrak{t}$

I'm working on the Malliaris-Shelah's recent result of $\mathfrak{p}=\mathfrak{t}$, but I'm more interested in what possible topological applications can derivate from this equality. Searching in ...
Alexei0709's user avatar
8 votes
0 answers
187 views

Specializing fat trees

The discussion is about trees of height $\omega_1$ that are not necessarily thin, namely, no cardinality constraints on the size of each level. A classcial theorem of Baumgartner states that it is ...
Otto's user avatar
  • 1,006
8 votes
0 answers
219 views

When can we force two frames to be homeomorphic?

Recall that if $M,N$ are two structures of the same type, then $M$ is $\mathcal{L}_{\infty,\omega}$ elementarily equivalent to $N$ precisely when $M$ and $N$ are isomorphic in some forcing extension. ...
Joseph Van Name's user avatar
8 votes
0 answers
300 views

Gleason’s Theorem for non-separable Hilbert spaces: On a theorem of Solovay

Gleason’s theorem plays an important role in the foundations of quantum mechanics. On the positive side it demonstrates how the probabilistic structure of quantum theory follows from its logical ...
Mohammad Golshani's user avatar